Why Does the Root Sum Calculation for a Polynomial's Derivative Confuse Many?

  • Thread starter Thread starter fayeshin
  • Start date Start date
  • Tags Tags
    Calculus
Click For Summary
The discussion centers on the confusion surrounding the root sum calculation for the polynomial 3x^5 - 250x^3 + 735x. The first derivative, 15x^4 - 750x^2 + 735, has roots that sum to 50, while the second derivative, 60x^3 - 1500x, has roots that sum to 25. The total sum of the x-coordinates of the critical points is thus 75, which contrasts with the initial assumption of 0. Participants express confusion over the method of calculating these sums, particularly regarding the treatment of positive and negative roots. Clarification is sought on why the sum does not account for all roots symmetrically.
fayeshin
Messages
5
Reaction score
0
The question is :"The polynomial 3x5-250x3+735x is interesting because it has the maximum possible number of relative extrema and points of inflection at integer lattice points for a quintic polynomial. What is the sum of the x-coordinates of these points?"

I think the answer is 0, but it's wrong.

My solution is :"The first derivative is 15x4-750x2+735, whose roots are -7,-1,1 and 7. The second derivative is 60x3-1500x,whose roots are 0,-5,5. Then, the sum is 0."

The HMMT solution is "The first derivative is 15x4-750x2+735, whose roots sum to 750/15=50. The second derivative is 60x3-1500x,whose roots sum to 1500/60=25, for a grand total of 75."

I really can't understand how it get the sum 50 and 25...Please help me.

Thanks...
 
Physics news on Phys.org
The only thing I can say is that they appear to be talking about the sum of x2 where x is a zero of the polynomial:
15x^4-750x^2+735= 15(x^2- 1)(x^2- 49)
so x2= 1 and 49 which add to 50 and
60x3-1500x= 60 x(x^2- 25)
so x2= 0 and 25 which add to 25.
 
HallsofIvy said:
The only thing I can say is that they appear to be talking about the sum of x2 where x is a zero of the polynomial:
15x^4-750x^2+735= 15(x^2- 1)(x^2- 49)
so x2= 1 and 49 which add to 50 and
60x3-1500x= 60 x(x^2- 25)
so x2= 0 and 25 which add to 25.

i think so...
but since we have +7 and -7 and so on...why isn't it 150?
 
thank you all the way...
 
Question: A clock's minute hand has length 4 and its hour hand has length 3. What is the distance between the tips at the moment when it is increasing most rapidly?(Putnam Exam Question) Answer: Making assumption that both the hands moves at constant angular velocities, the answer is ## \sqrt{7} .## But don't you think this assumption is somewhat doubtful and wrong?

Similar threads

Replies
4
Views
2K
Replies
2
Views
2K
Replies
2
Views
598
  • · Replies 5 ·
Replies
5
Views
3K
  • · Replies 10 ·
Replies
10
Views
5K
  • · Replies 175 ·
6
Replies
175
Views
26K
  • · Replies 67 ·
3
Replies
67
Views
15K
  • · Replies 1 ·
Replies
1
Views
3K
  • · Replies 1 ·
Replies
1
Views
3K
  • · Replies 7 ·
Replies
7
Views
4K